if a truck has a linear acceleration of 1.85 m/s2 and the wheels have an angular acceleration of 5.23 rad/s2, what is the diameter of the truck's wheels?

Answers

Answer 1

If a truck has a linear acceleration of 1.85 m/s² and the wheels have an angular acceleration of 5.23 rad/s², the diameter of the truck's wheels   0.71 m.

What is the difference between linear acceleration and angular acceleration?

Linear acceleration refers to the time rate of change of linear velocity, whereas angular acceleration refers to the time rate of change of angular velocity. This is the primary differential between linear and angular acceleration. Simply said, changes in an object's linear velocity with respect to time are represented by changes in linear acceleration.

The angular acceleration can be deduced immediately from the concept of α =ΔωΔt because the ultimate angular velocity and time are both provided.

The link between linear acceleration (a) and rotational acceleration is expressed as a = r×α . When the angular acceleration increases, so will the linear acceleration's strength. Increased wheel angular acceleration, for instance, denotes an accelerated vehicle.

Linear acceleration is the uniform acceleration caused by a moving body moving along a straight line. There are three equations that are essential in linear acceleration, depending on parameters like start and terminal velocities, displacements, times, and acceleration.

Given :

linear acceleration a = 1.85 m/s²

angular acceleration α  = 5.23 rad/s²

radius r = a/ α  = [tex]\frac{1.85}{5.23}[/tex] = 0.354 m

diameter  d = 2r = 2 × 0.354 = 0.71 m

diameter of the wheels is  0.71 m.

To know more about angular acceleration, visit:

https://brainly.com/question/29428475

#SPJ1


Related Questions

a 10.0-mf capacitor is fully charged across a 12.0-v bat- tery. the capacitor is then disconnected from the battery and connected across an initially uncharged capacitor with capacitance c. the resulting voltage across each capacitor is 3.00 v. what is the value of c?

Answers

The value of  uncharged capacitor in series with a 10.0-microfarad capacitor, given that each capacitor has a voltage of 3.00 volts, can be calculated using the formula for equivalent capacitance in series and  formula for charge on a capacitor. The value of c is approximately 4.00 microfarads.

To determine the value of c, which is [tex]1/Ceq = 1/C1 + 1/C2[/tex] . Initially, the 10.0-microfarad capacitor has a charge of [tex]Q = CV = (10.0 * 10^{-6 }F) * 12.0 V = 1.20 * 10^{-4} C[/tex].

When it is connected in series with uncharged capacitor with capacitance c,  charge is shared between the two capacitors. The charge on  10.0-microfarad capacitor is also equal to the charge on  uncharged capacitor, which is given by [tex]Q = (3.00 V) * C[/tex] .

Equating the two expressions for Q and solving for c, we get [tex]C = Q/3.00[/tex] [tex]V = (1.20 * 10^{-4 C}) / (3.00 V) = 4.00 * 10^{-5 F}[/tex]. Therefore,  value of c is approximately 4.00 microfarads.

To know more about equivalent capacitance, here

brainly.com/question/30905469

#SPJ4

A boy on a 1.9 kg skateboard initially at rest
tosses a(n) 8.0 kg jug of water in the forward
direction.
If the jug has a speed of 2.7 m/s relative to
the ground and the boy and skateboard move
in the opposite direction at 0.65 m/s, find the
boy’s mass.
Answer in units of kg.

Answers

Answer:

Approximately [tex]31.3\; {\rm kg}[/tex]. (Assuming the friction between the skateboard and the ground is negligible.)

Explanation:

The momentum [tex]p[/tex] of an object of [tex]m[/tex] and velocity [tex]v[/tex] is:

[tex]p = m\, v[/tex].

When the boy tossed the jug of water, the change in the momentum of the jug would be:

[tex]\Delta p(\text{jug}) = m(\text{jug}) \, (v(\text{jug}) - u(\text{jug}))[/tex], where:

[tex]m(\text{jug}) = 8.0\; {\rm kg}[/tex] is the mass of the jug;[tex]v(\text{jug}) = 2.7\; {\rm m\cdot s^{-1}}[/tex] is the velocity of the jug after the toss;[tex]u(\text{jug}) = 0\; {\rm m\cdot s^{-1}}[/tex] is the initial velocity of the jug, which was at rest before the toss.

Hence:

[tex]\begin{aligned}\Delta p(\text{jug}) &= m(\text{jug}) \, (v(\text{jug}) - u(\text{jug})) \\ &= (8.0)\, (2.7 - 0)\; {\rm kg\cdot m\cdot s^{-1}} \\ &= 21.6\; {\rm kg\cdot m\cdot s^{-1}}\end{aligned}[/tex].

Similarly, the change in the momentum of the skateboard would be:

[tex]\Delta p(\text{board}) = m(\text{board}) \, (v(\text{board}) - u(\text{board}))[/tex], where:

[tex]m(\text{board}) = 1.9\; {\rm kg}[/tex] is the mass of the board;[tex]v(\text{board}) =(-0.65)\; {\rm m\cdot s^{-1}}[/tex] is the velocity of the board after the toss;[tex]u(\text{board}) = 0\; {\rm m\cdot s^{-1}}[/tex] is the initial velocity of the board.

Note that the velocity of the board [tex]v(\text{board})\![/tex] after the toss is opposite to that of the jug. The sign of [tex]v(\text{board})[/tex] would be opposite to that of [tex]v(\text{jug})[/tex]. Since [tex]v(\text{jug})\![/tex] is positive, the value of [tex]v(\text{board})\!\![/tex] should be negative.

[tex]\begin{aligned}\Delta p(\text{board}) &= m(\text{board}) \, (v(\text{board}) - u(\text{board})) \\ &= (1.9)\, ((-0.65)- 0)\; {\rm kg\cdot m\cdot s^{-1}} \\ &= (-1.235)\; {\rm kg\cdot m\cdot s^{-1}}\end{aligned}[/tex].

Let [tex]m(\text{boy})[/tex] denote the mass of the boy. The velocity of the boy was initially [tex]u(\text{boy}) = 0\; {\rm m\cdot s^{-1}}[/tex] and would become [tex]v(\text{boy}) =(-0.65)\; {\rm m\cdot s^{-1}}[/tex] after the toss. The change in the velocity of the boy would be:

[tex]\Delta p(\text{boy}) = m(\text{boy}) \, (v(\text{boy}) - u(\text{boy}))[/tex].

Under the assumptions, the total changes in the momentum of this system (the boy, the skateboard, and the jug) should be [tex]0[/tex]. Thus:

[tex]\Delta p(\text{boy}) + \Delta p(\text{boy}) + \Delta p(\text{jug}) = 0[/tex].

Rearrange and solve for the mass of the boy:

[tex]\Delta p(\text{boy}) = -\Delta p(\text{jug}) - \Delta p(\text{board})[/tex].

[tex]\begin{aligned} m(\text{boy}) &= \frac{-\Delta p(\text{jug}) - \Delta p(\text{board})}{v(\text{boy}) - u(\text{boy})} \\ &= \frac{-(21.6) - (-1.235)}{(-0.65) - 0}\; {\rm kg} \\ &\approx 31.3\; {\rm kg}\end{aligned}[/tex].

a 2.70 kg ball is attached to a ceiling by a 1.35 m long string. the height of the room is 4.45 m. what is the gravitational potential energy of the ball relative to the ceiling?

Answers

The gravitational potential energy of the ball relative to the ceiling is 87.9 J.

The gravitational potential energy of an object of mass m at a height h above a reference level (in this case, the ceiling) is given by:

U = mgh

where g is the acceleration due to gravity.

In this problem, the ball is suspended from the ceiling by a string, so its height above the ceiling is the length of the string, minus the radius of the ball. Assuming the ball is a sphere with a radius of 0.135 m (half the length of the string), we can calculate its height above the ceiling as:

h = 4.45 m - 1.35 m + 0.135 m = 3.24 m

(Note that we subtract the length of the string from the height of the room, and add half the length of the string to account for the radius of the ball.)

Plugging in the given values, we get:

U = (2.70 kg)(9.81 m/s^2)(3.24 m)

U = 87.9 J

Therefore, the result is 87.9 J.

Learn more about the potential energy: https://brainly.com/question/24284560

#SPJ11

the plane is flying at 800 miles per hour. how far will the package travel horizontally during its descent?

Answers

The distance that a package will travel horizontally during its descent when a plane is flying at 800 miles per hour can be calculated using the following steps is 1600 miles.

What is the distance?

Determine the time taken for the package to hit the ground. We know that when an object is dropped from a certain height, it falls under the influence of gravity.

The acceleration due to gravity is 9.8 m/s². The formula for the time taken for an object to fall can be given by:

t = √(2h/g)

where, t is the time taken for the object to fall is the height from which the object was dropped g is the acceleration due to gravity.

We know that the distance traveled by the package horizontally can be given by d = vt

where, d is the distance traveled horizontally by the package v is the velocity of the planet is the time taken for the package to hit the ground.

Thus, the distance is 1600 miles.

Read more about Distance here:

https://brainly.com/question/15172156

#SPJ11

a 23.9 a current flows in a long, straight wire. find the strength of the resulting magnetic field at a distance of 58.3 cm from the wire.

Answers

The magnetic field at a distance of 58.3 cm from a long, straight wire carrying a 23.9 A current, the strength of the resulting magnetic field can be found using the equation B = μ0*I/2π*r, where B is the magnetic field strength, μ0 is the permeability of free space, I is current, and r is the distance.

Therefore, the strength of the magnetic field at 58.3 cm from the wire is B = 4π * 10-7 * 23.9/2π * 58.3 = 0.0067 N/Amp.


The magnetic field strength due to the current in the wire is caused by the current producing a magnetic field, which is a result of moving electric charges (electrons) in the wire. The strength of the magnetic field depends on the magnitude of the current and the distance from the wire.

As the current increases, the magnetic field strength increases; likewise, as the distance from the wire increases, the magnetic field strength decreases. The direction of the magnetic field can be determined using the right-hand rule.


The strength of the magnetic field can be used to calculate the force on a moving charged particle, F = q * v * B, where q is the charge of the particle, v is its velocity, and B is the magnetic field strength. By using this equation, the force acting on a charged particle due to the magnetic field at 58.3 cm from the wire can be found.

To know more about magnetic field refer here:

https://brainly.com/question/23096032#

#SPJ11

which satellite channel measures the temperature of the underlying surfaces (i.e., clouds, ocean, land)? group of answer choices visible infrared water vapor

Answers

Visible Infrared (IR) satellite channels measure the temperature of underlying surfaces. This includes clouds, oceans, and land.

IR channels work by detecting the amount of infrared radiation emitted from the Earth's surface. The intensity of the radiation is then converted into a digital number, which is displayed as a color on a satellite image. The higher the digital number, the warmer the surface temperature. This data can then be used to track changes in temperatures over time. The satellite channel that measures the temperature of the underlying surfaces is visible infrared. The surface temperature measurement is made possible by the difference in temperatures of objects in the infrared spectrum. An object's temperature and the level of radiation it emits have a direct correlation, and this is what visible infrared satellites use to take the temperature of the underlying surfaces. The visible infrared (VI) channel is used to estimate cloud cover and surface temperature. Infrared radiation from the surface of the earth is detected in this channel. The temperature of clouds, oceans, and land can be estimated using the visible infrared (VI) channel. It also provides data on how temperature changes with latitude and over time. Furthermore, the VI channel aids in the identification of cold and hot surfaces. Water vapor (WV) is another channel utilized in satellite imagery to observe the atmosphere's water vapor content. It enables meteorologists to forecast the occurrence of rainfall and other weather patterns. In general, satellite measurements assist in understanding Earth's weather and its impact on humans and the environment. These satellites help scientists to forecast severe weather, monitor weather changes over time, and analyze natural disasters. In addition, they assist in tracking the effects of climate change on the planet.

For more such questions on Satellites.

https://brainly.com/question/15168838#

#SPJ11

A load of 100g placed on a spiral spring, A extends it spring by 2cm when the same load is placed on spiral spring, B it extends it by 5cm, which spring has smaller stiffness

Answers

Spring A has smaller stiffness.Stiffness is a measure of the spring's resistance to deformation. The stiffer the spring, the more resistant it is to deformation. When a load is applied to a spring, it experiences deformation. Stiffness is a measure of how much force is required to deform the spring by a certain amount.

Springs with higher stiffness require more force to deform them than springs with lower stiffness.A load of 100g placed on a spiral spring, A extends its spring by 2cm, whereas the same load placed on spiral spring, B extends it by 5cm. The stiffness of a spring is inversely proportional to the amount of deformation it experiences. Spring B will be less stiff because it experiences more deformation than Spring A.Spring stiffness is measured in units of force per unit of length. The spring constant k is a measure of stiffness. It is defined as the amount of force required to extend the spring by one unit of length.The spring constant k can be calculated as follows:F = kxWhere F is the force applied, k is the spring constant, and x is the amount of deformation experienced by the spring. We can use this formula to calculate the spring constants for A and B:kA = F/x = 100g/(2/100) = 5000 N/mkB = F/x = 100g/(5/100) = 2000 N/mSpring A has a higher stiffness (5000 N/m) than spring B (2000 N/m) because it requires more force to deform it by the same amount. Hence, spring A has smaller stiffness.

For such more questions on stiffness:

brainly.com/question/30724864

#SPJ11

when the resistors are connected in 2 loops (first circuit of the video) the current through the resistors are 1 ma and 10ma. what is the current in the circuit before the junction that splits to the 2 resistors?

Answers

The circuit's initial current via the junction where the two resistors are separated is 11 mA. The current divides and simultaneously passes via both resistors in a paralleled resistor circuit using two resistors.

A battery and many capacitors are linked in series. The capacitors have a comparable amount of charge.

A battery and many capacitors are linked in series. The sum of the potential differences between each capacitor equals the current battery emf.

When two resistors having resistance R that are similar to one another are linked in series, the capacitive reactance is 2R.

Both negative and positive ions move charges whenever an electricity flows through with an ionic liquid like salty water. Energy is measured in electron-volts.

To know more about current click here

brainly.com/question/30832987

#SPJ4

if a certain passenger arrives at the station at a time uniformly distributed between 7 and 8 a.m. and then gets on the first train that arrives, what proportion of time does he or she go to destination a?

Answers

The probability that the passenger will get on the first train that arrives is the same as the probability that they will arrive at the station between 7 and 8 a.m., which is 1/2.

The uniform distribution is a type of probability distribution where all outcomes are equally likely. In this case, the passenger arrives at the station at a time that is uniformly distributed between 7 and 8 a.m. Therefore, the probability that the passenger will get on the first train that arrives is the same as the probability that they will arrive at the station between 7 and 8 a.m., which is 1/2.
In other words, the probability that the passenger will go to destination A is 1/2. This is because the probability that they will arrive between 7 and 8 a.m. and get on the first train that arrives is the same as the probability that they will arrive between 7 and 8 a.m., which is 1/2.

Therefore, the proportion of time the passenger goes to destination A is 1/2. This is because the probability of them getting on the first train that arrives is the same as the probability of them arriving between 7 and 8 a.m., which is 1/2.

For more such questions on Probability distribution.

https://brainly.com/question/14210034#

#SPJ11

if each charge has two field lines per unit of charge (q), what is the ratio of the total positive (red) charge to the total negative (blue) charge?

Answers

The ratio of total positive charge (red) to total negative charge (blue) is 1:1. This is because for each unit of charge (q), there are two field lines, one for the positive charge and one for the negative charge.

What are field lines?

Field lines are a visual tool used to represent the direction and strength of an electrical field. The direction of a field line shows the direction of the force that a positive test charge would experience if it were placed at that point in the field. Meanwhile, the density of the field lines indicates the strength of the electric field.

Since each charge has two field lines per unit of charge (q), it means that the total number of field lines is proportional to the total charge. If there are equal numbers of field lines coming from both the positive and negative charges, it means that the ratio of the total positive charge to the total negative charge is 1:1.

Read more about electricity:

https://brainly.com/question/24786034

#SPJ11

a rear window defroster consists of a long, flat wire bonded to the inside surface of the window. when current passes through the wire, it heats up and melts ice and snow on the window. for one window the wire has a total length of 11.0 m , a width of 1.8 mm , and a thickness of 0.11 mm . the wire is connected to the car's 12.0 v battery and draws 7.5 a . part a what is the resistivity of the wire material? express your answer with the appropriate units.

Answers

The resistivity of the wire material can be calculated using Ohm's Law, which states that V=IR, or voltage = current multiplied by resistance. Therefore, the resistivity of the wire material is [tex]2.87 \times 10^{-8} \Omega m[/tex].

Resistivity of wire is given as ρ=RA/L where R is the resistance of wire, A is the cross-sectional area of the wire, L is the length of the wire.

The formula to calculate the resistance of wire from Ohm's Law is given by R=V/I where V is the voltage, I is the current.

Substituting the given values: V = 12.0 V, I = 7.5 A.

Therefore, R=V/I=12.0 / 7.5 = 1.6 Ω

From the formula of resistivity:

[tex]\rho=\dfrac{RA}{L}\\R=\dfrac{ρL}{A}[/tex]

Substituting the given values: R = 1.6 Ω, L = 11.0 m and calculating the area:

[tex]A = (1.8 \times 10^{-3} m) (0.11 \times 10^{-3} m)\\ = 0.198 \times 10^{-6} m²[/tex]

Therefore,

[tex]\rho = RA/L\\= \dfrac{R \times A}{ L}\\= \frac{1.6 \times 0.198 \times 10^{-6}}{ 11.0}\\ = 2.87 \times 10^{-8 } \Omega m[/tex]

Therefore, the resistivity of the wire material is [tex]2.87 \times 10^{-8 } \Omega m[/tex].

For more details on resistivity, click on the below link:

https://brainly.com/question/30799966

#SPJ11

a point charge of 9.00 109 c is located at the origin. how much work is required to bring a positive charge of 3.00 109 c from infi nity to the location x 30.0 cm?

Answers

The amount of work required to bring a positive charge of 3.00 109 c from infinity to the location x 30.0 cm can be calculated using the formula W = qV, where W is the work, q is the charge, and V is the potential difference.

The potential difference for this situation is equal to the electric potential at 30.0 cm, which is equal to the electric potential from the 9.00 109 c point charge at the origin.

The work needed is equal to the charge multiplied by the potential difference, so W = qV = (3.00 109 c)(9.00 109 c/30.0 cm) = 9.00 108 c2/cm. This is the amount of work required to bring a positive charge of 3.00 109 c from infinity to the location x 30.0 cm.

Know more about work here

https://brainly.com/question/12469807#

#SPJ11

Research Galileo's work on falling bodies What did he wanted to demonstrate?What arguments did he use to prove that he was right?did be used experiments logic finding of other scientists or other approaches

Answers

Galileo Galilei conducted experiments on falling bodies to demonstrate that the rate of fall is independent of an object's mass.  Galileo argued that if heavier objects did indeed fall faster, then two objects of different masses tied together would fall at an intermediate speed, which he found was not the case.

He used various methods to prove his point, including rolling balls down inclined planes, dropping weights from towers, and measuring the times of fall. He also used logic and mathematical reasoning to support his conclusions. Galileo's work marked a significant shift from traditional Aristotelian physics to the empirical approach of modern science.

To know more about modern science, here

brainly.com/question/30252960

#SPJ4

a heavy fish (mass 4 kg) is swimming with a speed of 1.5 m/s. he then swallows a light fish (1.2 kg), swimming toward him with a speed of 3.0 m/s. what is the velocity of the larger fish after he had his meal? (you can neglect water resistance.)

Answers

The velocity of the larger fish after the meal is zero.

We can use the law of conservation of momentum, which states that the total momentum of a closed system remains constant. Before the light fish is swallowed, the total momentum is,

p1 = m1v1 + m2v2

where m1 = 4 kg, v1 = 1.5 m/s (velocity of the heavy fish), m2 = 1.2 kg, and v2 = -3.0 m/s (negative because the light fish is swimming toward the heavy fish).

p1 = (4 kg)(1.5 m/s) + (1.2 kg)(-3.0 m/s)

p1 = 0 kg m/s

After the light fish is swallowed, the two fish become one system. Let the velocity of the larger fish after the meal be v.

The total momentum of the system after the meal is,

p2 = (m1 + m2)v

By the law of conservation of momentum, p1 = p2,

0 kg m/s = (4 kg + 1.2 kg) v

Solving for v,

v = 0 m/s

To know more about speed, here

brainly.com/question/31080132

#SPJ4

A droplet of ink in an industrial ink-jet printer carries a charge of 2.1×10?10C and is deflected onto paper by a force of 3.2×10?4N. Find the strength of the electric field (E=F/q) required to produce this force. Express your answer to two significant figures and include the appropriate units.

Answers

The electric field strength needed to generate this force is roughly 1.5 x 106 N/C

We know that the strength of the electric field is defined as,E = F/qWhere,E = Electric field strength,F = Force on the droplet of ink,q = Charge on the droplet of ink.Therefore, putting the given values, we get: E = (3.2 × 10⁻4 ) / (2.1 ×10-4 ) = 1.5 × 10⁶ N/C.

Thus, the strength of the electric field required to produce the force is 1.5 ×10⁶ N/C (two significant figures). Therefore, the final answer is 1.5 × 10⁶ N/C.

To know more about electric field strength  click on below link :

https://brainly.com/question/28227168

#SPJ11

you have two flat metal plates, each of area 2.00 m2, with which to construct a parallel-plate capacitor. if the capacitance of the device is to be 1.00 f, what must be the separation between the plates?

Answers

The capacitance of the device is to be 1.00 f, the separation distance between the plates is 1.77 × 10⁻¹³ m.

We have two flat metal plates of area 2.00 m² each with which to construct a parallel-plate capacitor. If the capacitance of the device is to be 1.00 F

Given:

Area of each plate = 2.00 m²

Capacitance of the device = 1.00 F

We know that the capacitance of a parallel-plate capacitor is given by:

C = εA/d

Where C is the capacitance of the parallel plate capacitor, ε is the permittivity of the material between the plates, A is the area of the plate, and d is the separation distance between the plates.

Rearranging this equation we get:

d = εA/C

Now, to find the separation distance, we need to know the permittivity of the material between the plates. The permittivity of a vacuum is 8.85 × 10⁻¹² F/m.

Since the question doesn't specify the permittivity of the material between the plates, we will assume it to be a vacuum. So,

ε = 8.85 × 10⁻¹² F/m²

Substituting the values of ε, A, and C, we get:

d = εA/C= (8.85 × 10⁻¹² F/m²) × (2.00 m²) / (1.00 F)

= 17.7 × 10⁻¹² m²/F /F= 17.7 × 10⁻¹² m

= 1.77 × 10⁻¹³ m

Therefore, the separation distance between the plates is 1.77 × 10⁻¹³ m.

For more such questions on distance , Visit:

https://brainly.com/question/26550516

#SPJ11

europa, one of the moons of jupiter, was discovered by galileo in 1610. europa has a circular orbit of radius 6.708 105 km and period 3.551 days. find the mass of jupiter.

Answers

Therefore, the mass of Jupiter is approximately 1.898 × 1027 kg.

The mass of Jupiter can be calculated using the equation M = (4π2 r3)/(G P2), where M is the mass of Jupiter, r is the orbital radius of Europa (6.708 105 km), G is the gravitational constant (6.674 × 10-11 m3 kg-1 s-2), and P is the orbital period of Europa (3.551 days).

The circular orbit of Europa is given as, r = 6.708 × 105 km. The period of Europa is given as, T = 3.551 days are supposed to calculate the mass of Jupiter. In order to calculate the mass of Jupiter, we need to use Kepler's 3rd law. Kepler's 3rd law is given as, T2 = (4π2/GM) × r3 where T is the period of orbit, G is the gravitational constant, M is the mass of the planet, and r is the radius of the orbit.

By rearranging the above formula we get, M = (4π2r3) / (GT2)Substituting the given values, we get, M = (4π2 × (6.708 × 105)3) / ((6.67430 × 10-11) × (3.551 × 24 × 60 × 60)2) ≈ 1.898 × 1027 kg. Therefore, the mass of Jupiter is approximately 1.898 × 1027 kg.

You can read more about Jupiter at https://brainly.com/question/2715802#:

#SPJ11

What is the maximum ramp angle that still allows the crate to remain at rest? (Make sure the coefficient of friction is 0.7.) .
Mass (m) = 300kg

Answers

The highest ramp angle at which the crate can still be at rest is roughly 35.5 degrees.

To determine the maximum ramp angle that still allows the crate to remain at rest, you need to consider the balance of forces acting on the crate. When the crate is on the verge of slipping, the frictional force is equal to the component of gravitational force acting parallel to the ramp.

Given that the coefficient of friction (µ) is 0.7, you can use the formula for the frictional force:

Frictional force (F_friction) = µ * Normal force (F_N)

The normal force acting on the crate is the component of gravitational force acting perpendicular to the ramp, which can be calculated as:

F_N = m * g * cos(θ)

The gravitational force acting parallel to the ramp can be calculated as:

F_gravity_parallel = m * g * sin(θ)

At the maximum angle, the frictional force will be equal to the gravitational force acting parallel to the ramp:

µ * F_N = F_gravity_parallel

Now, substitute the known values:

0.7 * (m * g * cos(θ)) = m * g * sin(θ)

Since the mass (m) and gravitational acceleration (g) are the same on both sides of the equation, they can be canceled out:

0.7 * cos(θ) = sin(θ)

To find the maximum angle (θ), you can use the arctangent function:

θ = arctan(0.7)

θ ≈ 35.5 degrees

So, the maximum ramp angle that still allows the crate to remain at rest is approximately 35.5 degrees.

To know more about frictional force click on below link :

https://brainly.com/question/1714663#

#SPJ11

If a 20-kilogram anvil is held 3 meters what is the potential energy?

Answers

The potential energy (PE) of an object is given by the formula:

PE = mgh

where m is the mass of the object, g is the acceleration due to gravity (9.8 m/s^2 on Earth), and h is the height of the object above some reference point (in this case, the ground).

Substituting the given values, we get:

PE = (20 kg) x (9.8 m/s^2) x (3 m) = 588 J

Therefore, the potential energy of the 20-kilogram anvil held 3 meters above the ground is 588 joules (J).

learn more about potential energy here:

https://brainly.com/question/24284560

#SPJ4

how much work is done lifting a 15 pound object from the ground to the top of a 30 foot building if the cable used weighs 2 pounds per foot

Answers

The amount of work required to lift a 15 pound object from the ground to the top of a 30 foot building if the cable used weighs 2 pounds per foot is 1050 foot-pounds.

In order to solve the problem, we can use the formula W = Fd. where, W is the work done, F is the force required and d is the distance covered by the object while being lifted or moved.

So, we have to first calculate the force required to lift the object. Let us assume the force required is F, then

F =  weight of object + weight of cable

F = 15 + 2 * 30

F = 75 pounds

Therefore, the force required to lift the object is 75 pounds. Now, we can calculate the work done as follows:

W= Fd

W = 75 * 14

W = 1050 foot-pounds

Therefore, the amount of work required to lift a 15 pound object from the ground to the top of a 30 foot building if the cable used weighs 2 pounds per foot is 1050 foot-pounds.

Learn more about work of move at https://brainly.com/question/28356414

#SPJ11

at a particular instant, a hot air balloon is 210 m in the air and descending at a constant speed of 3.5 m/s. at this exact instant, a girl throws a ball horizontally, relative to herself, with an initial speed of 21 m/s. when she lands, where will she find the ball? ignore air resistance. (find the distance, in meters, from the girl to the ball.)

Answers

The ball which is thrown with a speed of 21 m/s, travels a distance of 129.99 m in the horizontal direction.

Therefore, the vertical component of the ball's motion will be determined by the force of gravity and the initial vertical speed of the balloon.

We can use the following kinematic equation to determine how long it takes for the ball to fall to the ground:

h = ut + 1/2 * g * t^2

where h is the initial height of the ball (equal to the height of the balloon which is 210 m).

u is the initial velocity of the ball in the vertical direction which is 3.5 m/s.

g is the acceleration due to gravity (approximately 9.8 m/s^2),

and t is the time it takes for the ball to fall to the ground.

Plugging in the values we know, we get:

210 = 3.5 * t + 1/2 * 9.8 * t^2

4.9 t^2 + 3.5 t - 210 = 0

t = 6.19 seconds

Now we can use the time it takes for the ball to fall to the ground to determine how far it travels horizontally, given its initial horizontal velocity of 21 m/s. We can use the following equation:

d = v * t

where d is the horizontal distance traveled by the ball, v is its initial horizontal velocity, and t is the time it takes to fall to the ground (which we just calculated).

Plugging in the values we know, we get:

d = 21 * 6.19

d ≈ 129.99 meters

Therefore, the girl will find the ball approximately at a distance of 129.99 meters away from her when she lands after throwing the ball horizontally.

Learn more about the kinematic equation:

https://brainly.com/question/28712225

#SPJ11

a girl weighing 455 n jumps from a tree, and her center of mass falls a vertical distance of 1.50 m. find the impulse necessary to bring her to rest.

Answers

The impulse necessary to bring her to rest is zero (0 Ns). Taking into account that the girl's momentum was maintained even as she fell, and since she started from rest, her final momentum should also be zero. So no additional push is needed beyond what gravity provides.

To find the impulse necessary to bring the girl to rest, we need to use the principle of conservation of momentum, which states that the total momentum of a system is conserved in the absence of external forces. In this case, we can assume that the girl is initially at rest, so her initial momentum is zero.

When the girl jumps from the tree, she is subject to the force of gravity, which causes her to accelerate downwards. We can use the equation for the gravitational potential energy to find the work done by gravity:

[tex]W = mgh[/tex]

Where W is the work done by gravity, m is the mass of the girl, g is the acceleration due to gravity, and h is the vertical distance that the center of mass falls.

Plugging in the given values, we get:

[tex]W = (455 N)(1,50 m)(9,81 m/s^2) \\W= 6.717,08 J[/tex]

This work done by gravity is equal to the change in kinetic energy of the girl, which can be expressed as the impulse required to bring her to rest:

J = ΔK

[tex]J= -mv[/tex]

where J is the impulse, ΔK is the change in kinetic energy, m is the mass of the girl, and v is her final velocity. Since the girl comes to a stop, her final velocity is zero, so we can simplify the equation to:

[tex]J = mv[/tex]

Plugging in the given mass and solving for the impulse, we get:

[tex]J = (455 N)(-0 m/s) \\J = 0 Ns[/tex]

Therefore, the impulse necessary to bring the girl to rest is zero.

Learn more about the impulse at: https://brainly.in/question/1383595

#SPJ11

your car is accelerating to the right from a stop.for the steps and strategies involved in solving a similar problem, you may view a

Answers

To solve the given problem, it is important to understand the concept of acceleration and the forces acting on the car. The acceleration of a car is the rate at which its velocity changes over time.

The forces acting on the car can be divided into two components: the force of friction between the tires and the road, and the force of gravity acting on the car.

The force of friction depends on the nature of the road surface and the type of tires on the car. The force of gravity depends on the mass of the car and the gravitational acceleration.

It is given that the car is accelerating to the right from a stop. This means that the car is moving in the positive x-direction with an increasing velocity.Identify the forces acting on the car: The forces acting on the car are the force of friction and the force of gravity. The force of friction is acting in the opposite direction to the motion of the car and is given by f = μN, where μ is the coefficient of friction and N is the normal force acting on the car. The force of gravity is acting in the downward direction and is given by Fg = mg, where m is the mass of the car and g is the gravitational acceleration.Analyze the motion of the car using the concepts of force and acceleration. The net force acting on the car is given by Fnet = ma, where a is the acceleration of the car. From Newton's second law, we can write Fnet = f - Fg = ma. Solving for a, we get a = (f - Fg)/m.Calculate the acceleration of the car by substituting the values of f, Fg, and m in the above equation, we get a = (μN - mg)/m. The normal force acting on the car is equal to the weight of the car, which is given by N = mg. Substituting this value in the above equation, we get a = (μ - g)/m. This is the expression for the acceleration of the car.

Therefore, a = (μ - g)/m is the expression for the acceleration of the car.

To know more about acceleration click here:

https://brainly.com/question/30660316

#SPJ11

which is the proper order of structures through which light must pass in order to perceive and image?

Answers

The proper order of structures through which light must pass in order to perceive and image is cornea, aqueous humor, lens, vitreous humor, retina.

These are the five main structures of the human eye that enable vision, including light perception and imaging. Let's delve into each of these structures.

Cornea: The clear, protective outer layer of the eye is the cornea. The cornea has two purposes: to shield the inner eye from harm and to help focus light on the retina at the back of the eye.

The cornea's curved shape bends light waves as they enter the eye, assisting in their concentration.

Aqueous humor: This is a liquid that flows throughout the front of the eye, nourishing and removing waste from its surrounding tissues.

It aids in the maintenance of normal eye pressure, and if this pressure becomes too high, it can lead to glaucoma.

Lens: The lens' job is to concentrate light onto the retina. It's a transparent structure with a biconvex (lens-like) shape that varies in thickness.

It is supported by ciliary muscles that allow it to alter shape when we focus on things at different distances.

Vitreous humor: This gel-like substance fills the eye's posterior (rear) cavity, providing it with structural stability and helping it to maintain its form. It also assists in light refraction.

Retina: This is a thin layer of tissue lining the rear of the eye. The retina's photoreceptor cells, or rods and cones, are sensitive to light.

The retina converts light energy into neural signals that are transmitted to the brain via the optic nerve, which is located behind the eye. The brain translates these signals into images, allowing us to see.

What we see when we open our eyes is formed by light. In order to perceive an image, light must pass through a series of structures in the eye.

The cornea, aqueous humor, lens, vitreous humor, and retina are the five main structures of the human eye that enable vision, including light perception and imaging.

to know more about light refer here:

https://brainly.com/question/29994598#

#SPJ11

a material has temperature coefficient of resistance (alpha) of 3.9 x 10^-3. if the material has a resistance of 23 ohms at a temperature of 20 c, what is the resistance of this material at a temperature of 50 c?

Answers

The resistance of the material at a temperature of 50°C is approximately 25.791 Ω.

We can use the formula for temperature dependence of resistance to solve this problem:

R2 = R1 [1 + α(T2 - T1)]

where R1 is the resistance at temperature T1, R2 is the resistance at temperature T2, and α is the temperature coefficient of resistance.

Plugging in the given values, we get:

R2 = 23 Ω [1 + (3.9 x 10⁻³/°C)(50°C - 20°C)]

Simplifying, we get:

R2 = 23 Ω [1 + (3.9 x 10^-3/°C)(30°C)]

R2 = 23 Ω [1 + 0.117]

R2 = 23 Ω [1.117]

R2 = 25.791 Ω

Therefore, the resistance of the material is approximately 25.791 Ω.

Learn more about Resistance:

#SPJ11

the electric field 0.300 m from a very long uniform line of charge is 850 n/c . part a how much charge is contained in a section of the line of length 1.70 cm ? express your answer in coulombs.

Answers

The charge in the section of the line of length 1.70 cm is:$$Q = (1.70 × 10⁻² m) * (2.16 × 10⁻⁵ C/m) = 1.277 × 10⁻⁷ C

The electric field 0.300 m from a very long uniform line of charge is 850 n/c. How much charge is contained in a section of the line of length 1.70 cm? The answer is 1.277 × 10⁻⁷ C. Explanation: To begin, let's consider the electric field due to an infinite line of charge. The electric field generated by a uniformly charged infinite line of charge is given by:$$E = \frac{λ}{2πεr}$$where, E is the electric field, λ is the linear charge density (charge per unit length), r is the distance from the wire, and ε is the permittivity of free space. To begin with, we can rearrange the equation for electric field:$$λ=\frac{2πεrE}{l}$$Where, l is the length of the line section of interest, E is the electric field at the distance r from the line of charge, and λ is the linear charge density. Now we can plug in the given values:$$(1.70 cm)λ = Q$$$$λ=\frac{2πεrE}{l}$$λ = (2π * 8.85 × 10⁻¹² F/m) * (0.300 m) * (850 N/C) / (0.0170 m)λ = 2.16 × 10⁻⁵ C/mSo, the charge in the section of the line of length 1.70 cm is:$$Q = (1.70 × 10⁻² m) * (2.16 × 10⁻⁵ C/m) = 1.277 × 10⁻⁷ C$$Therefore, 1.277 × 10⁻⁷ C.

Learn more about Charge line of length

brainly.com/question/14433096

#SPJ11

a car starts from rest and moves around a circular track of radius 47.0 m. its speed increases at the constant rate of 0.600 m/s2. (a) what is the magnitude of its net linear acceleration 15.0 s later?

Answers

The magnitude of the net linear acceleration of the car 15.0 seconds later is 5.08 m/s2. This is because acceleration is the rate of change of velocity, and the car's velocity is increasing at a constant rate of 0.600 m/s2.

To calculate the magnitude of the net linear acceleration, we must use the equation a = v2/r, where a is the acceleration, v is the velocity, and r is the radius of the circular track. Since the velocity of the car is increasing at a constant rate of 0.600 m/s2, we can calculate the velocity of the car after 15.0 seconds using the equation v = v0 + at, where v0 is the initial velocity (0 m/s in this case), a is the acceleration (0.600 m/s2), and t is the time (15.0 seconds).
Thus, the velocity of the car after 15.0 seconds is 9.00 m/s. Now, we can plug this velocity, along with the radius of the circular track (47.0 m), into the equation a = v2/r to calculate the magnitude of the net linear acceleration:
a = (9.00 m/s)2/47.0 m = 5.08 m/s2

Therefore, the magnitude of the net linear acceleration of the car 15.0 seconds later is 5.08 m/s2.

For more such questions on Linear acceleration.

https://brainly.com/question/12996537#

#SPJ11

two identical carts, both of mass 0.5 kg are moving towards each other, each with a speed of 1.5 m/s. after they collide, what will be their velocities?

Answers

After the collision, the first cart moves to the left with a velocity of -1.5 m/s and the second cart moves to the right with a velocity of 1.5 m/s.

The velocities of the two carts after collision can be determined using the conservation of momentum principle. Momentum is defined as the product of an object's mass and velocity. Given,Mass of each cart, m = 0.5 kg, Initial velocity of each cart, u = 1.5 m/s, Initial momentum of each cart, p = mu.

After collision, velocity of the carts = v. Using the law of conservation of momentum;

mu + mu = mv + mv⇒ 2mu = 2mv⇒ u = v

Momentum before collision = Momentum after collision (conservation of momentum)

∴ 0.5 × 1.5 + 0.5 × (-1.5) = 0.5v1 + 0.5v2

On solving, we get,v1 = -1.5 m/sv2 = 1.5 m/s

Therefore after the collision, the first cart moves to the left with a velocity of -1.5 m/s and the second cart moves to the right with a velocity of 1.5 m/s.

More on velocity: https://brainly.com/question/30470329

#SPJ11

a resistor is connected across the terminals of a 12 v battery, which delivers 1.47 j of energy to the resistor in 6.50 hours. what is the resistance of the resistor

Answers

The resistance of the resistor is 2.8 ohms.

The resistance of the resistor is calculated using the formula Power = Voltage x Current, or P = V x I.

Plugging in the given values, we get:

1.47 J = 12 V x I x 6.50 hours

Rearranging to solve for I, we get:

I = 1.47 J / (12 V x 6.50 hours)

Then, using Ohm's law (V = I x R) we can solve for R:

R = 12 V / I

Substituting in the value of I, we get:

R = 12 V / (1.47 J / (12 V x 6.50 hours))

Therefore, the resistance of the resistor is 2.8 ohms.

Resistance is the opposition that a substance offers to the flow of electric current

To know more about resistance, refer here:

https://brainly.com/question/29427458

#SPJ11

if the magnitude of the drift velocity of free electrons in a copper wire is 7.94 10-4 m/s, what is the electric field in the conductor?

Answers

The electric field in the copper wire is approximately 0.0227 V/m.

The drift velocity of free electrons in a copper wire is related to the electric field in the conductor by the following formula,

v_d = (e * E * τ) / m

where v_d is the drift velocity, e is the charge of an electron, E is the electric field strength, τ is the relaxation time of the electrons, and m is the mass of an electron.

Solving for E, we get,

E = (m * v_d) / (e * τ)

Substituting the given values for copper, we get,

E = (9.11 x 10^-31 kg * 7.94 x 10^-4 m/s) / (1.60 x 10^-19 C * 2.0 x 10^-14 s)

E = 0.0227 V/m (rounded to four significant figures)

To know more about electric field, here

brainly.com/question/12911661

#SPJ4

Other Questions
cost control involves containing construction costs within the budget established by the cost estimate. question 3 options: true false g what is the relationship between the wavelength of light, its color, and the energy of its photons? an evolutionary splitting event that gives rise to a distinct daughter species but with the persistence of the ancestral form is called Which table of values represents the linear function y=4x+1 which options show a health benefit of muscular endurance and muscular strength? (select all that apply.) physical fitness may be achieved by engaging in a moderately intense aerobic activity at a frequency of SummaryCan Our Surrounding Environment Really AffectUs? Can you make some observations about the picture and analyze it pls Help me with the even numbers. 2,4,6,and8 in a causal study of the effect of shelf placement on sales of a brand of cereal, which is the dependent variable? group of answer choices where the cereal was placed on the shelf sales of the cereal concomitant variation of the cereal none of the above every friday afternoon, li's company encourages people to wear casual clothes to work. this is an example of the organizaton's: 7.3 Puzzle time What Kind of Ship Can Last Forever? which has a greater (magnitude of) linear momentum: a 1000 kg truck moving at 30 mph, or a 500 kg car moving at 60 mph? which precautions are shared with family members who will be assisting the patient with application of nitro patches ANSWER RIGHT NOW WILL GIVE BRAINLIESTWrite a paragraph that creates a silly mood. Qu tipo de evento se promociona en estecartel publicitario? question 1 in addition to pregnant women, fetuses, and neonates, another subpart of the hhs regulations provides additional protections for which of the following vulnerable populations? college students the elderly adults with decisional impairments prisoners question 2 according to the federal regulations, research is eligible for exemption, if: the research falls into one of eight categories of research activity described in the regulations. all the subjects are adults and the risk is minimal. the researcher is experienced in the field of inquiry. participation in the research will involve ten minutes or less of the subjects' time. question 3 according to federal regulations, the expedited review process may be used when the study procedures pose: a minor increase over minimal risk and the sponsor needs approval before the next irb meeting. no more than minimal risk and the research activities fall within regulatory categories identified as eligible. any level of risk, but all the subjects are adults. more than minimal risk, but the study replicates previously approved research. question 4 which of the following statements about the relationship between an institution and the institution's irb(s) is correct? officials of the institution may overrule an irb approval. department chairs can overturn an irb disapproval. officials of the institution may overturn an irb disapproval. institutional priorities take precedence over all irb determinations. question 5 continuing review of an approved and ongoing study posing more than minimal risk that was initially approved by a convened irb: must occur within 12 months of the approval date. must be conducted by a convened irb. is limited to review of unanticipated problems. is not required unless additional risks have been identified. I'm learning probability in geometry but haven't learned it for percentage. Can someone help me? true or false: the robinson-patman act limits your ability to offer discounts to the chain supermarkets while leaving the price high for the smaller stores. Which has more kinetic energy:a. A compact car going 70 MPH or a tractor trailer going 15 MPH?b. An SUV going 30 MPH or a pickup truck going 30 MPH?c. A school bus going 15 MPH, an SUV going 35 MPH, or a compact car going 45 MPH?